0 Daumen
54 Aufrufe

$$\text{Seien } X_1,...,X_n \text{ unabhängige und Poisson-verteilte Zufallsvariablen mit Parameter } \lambda \geq 0.$$

$$ \text{ Die Zähldichte der Poisson-verteilung mit Parameter } \lambda \text{ ist gegeben durch}$$

$$ p(k)= \frac {\lambda ^k} {k!} exp(-\lambda) \text{  für } k \in \mathbb N_0.$$

$$ \text{Außerdem gilt } \mathbb E(Poi_{\lambda})=\lambda \text{ und } \mathbb V ar(Poi_{\lambda})=\lambda$$

$$\text{ a) Zeigen Sie, dass der Maximum-Likelihood-Schätzer }\hat{\lambda}_n \text{ von } \lambda \text{ basierend auf } X_1,...,X_n\text{ durch }$$
$$  \hat{\lambda}_n=\frac {1} {n} \sum \limits_{i=1}^{n} X_i \text{ gegeben ist. }$$

$$\text{Bemerkung: Es genügt zu zeigen, dass es sich um ein lokales Maximum handelt}$$

$$\text{ b) Zeigen Sie, dass } \hat{\lambda}_n \text{ ein erwartungstreuer und stark konsistenter Schätzer für }\lambda \text{ ist.}$$

$$\text{ c) Es gelte }\lambda \gt0. \text{ Weisen Sie nach, dass zudem }\sqrt{n}(\hat{\lambda}_n-\lambda) \xrightarrow[\text{}]{\text{D}} N_{(0,\lambda)} \text{ für } n \rightarrow \infty \text{ gilt.}$$

$$\text{ d) Sei } \alpha \in (0,1). \text{ Geben Sie, basierend auf dem Resultat aus c), ein asymptotisches }$$

$$ (1-\alpha) \text{-Konfidenzintervall für }\lambda \text{ für die richtigen Parameter } R_\lambda =\{\lambda\} \text{ an.}$$


ich bereite mich gerade auf die Klausur vor und wäre sehr dankbar über jede Hilfe, da es leider keine Musterlösung gibt...

Avatar von

Ein anderes Problem?

Stell deine Frage

Willkommen bei der Mathelounge! Stell deine Frage einfach und kostenlos

x
Made by a lovely community